Solving Economics Derivatives: Complementary Inputs & APL Minimization

  • Thread starter Thread starter annalynne
  • Start date Start date
  • Tags Tags
    Derivatives
Click For Summary
The discussion centers on solving an economics problem involving the average productivity of labor (APL) and its minimization, as well as the relationship between capital (K) and labor (L) as complementary inputs. The user seeks assistance in determining the level of L that minimizes APL and in proving that an increase in K leads to higher marginal productivity of labor (MPL) and APL. Calculus is employed to derive APL and MPL, with specific numerical examples provided to illustrate the relationship between K and L. A correction is noted regarding the derivative of APL, emphasizing the importance of proper mathematical manipulation. The thread concludes with a request for further clarification and assistance on the calculus involved.
annalynne
Messages
1
Reaction score
0
Hey everyone! This is an Economics question that's stumping me; but it requires a lot of calculus. Any help would be appreciated!

Homework Statement


Y = A[K^α][L^(1-α)/2][H^(1-α)/2] where 0 < α < 1.

A) At what level of L is APL (average productivity of labour) minimized?
B) Show K and L are complementary inputs in that more capital increases MPL and APL.

Homework Equations



MPL = marginal productivity of labor = ∂Y/∂L
APL = average productivity of labor = Y/L

The Attempt at a Solution



A) APL = Y/L = A[K^α][L^(α/2)][H^(1-α)/2]
minimize: therefore, take derivative and set to 0
∂APL/∂L = α/2[A][K^α][L^(α-1)/2][H^(1-α)/2] = 0
(I have no idea how to set solve this; would it also be possible to use the quotient rule to solve ∂APL/∂L from APL=A[K^α][L^(1-α)/2][H^(1-α)/2]/L ?)

B) prove that an increase in K causes and increase in MPL and APL
Y = A[K^α][L^(1-α)/2][H^(1-α)/2]
condition 1: assume A = 1, α = 0.6, K = 2, L = 3, H = 2
MPL = ∂Y/∂L = (1-α)/2 [A][K^α][L^(α/2)][H^(1-α)/2]
MPL = (0.25)(1)(1.41)(1.32)(1.19)
MPL = 0.55
APL = Y/L = A[K^α][L^(α/2)][H^(1-α)/2]
APL = (1)(1.41)(1.32)(1.19)
APL = 2.21

condition 2: assume A = 1, α = 0.6, K = 4, L = 3, H = 2
MPL = ∂Y/∂L = (1-α)/2 [A][K^α][L^(α/2)][H^(1-α)/2]
MPL = (0.25)(1)(2)(1.32)(1.19)
MPL = 0.79
APL = Y/L = A[K^α][L^(α/2)][H^(1-α)/2]
APL = (1)(2)(1.32)(1.19)
APL = 3.14

therefore, K and L are complements because APL and MPL increase as K increases

Any help would be appreciated! Thanks guys!
 
Physics news on Phys.org
For the first one, I think you have made a small mistake. If you take L^(1-α)/2 and divide it by L, you get L^(-α)/2 instead of L^(α)/2. Then taking the derivative you will get something like
1/L^(-α-1)
which can only become zero if α < -1.

For the second one, you could take the derivative w.r.t K and show that it is positive everywhere (I don't know if you are allowed to plug in numbers, in mathematics that usually means checking for a specific case instead of proving it generally).
 
Question: A clock's minute hand has length 4 and its hour hand has length 3. What is the distance between the tips at the moment when it is increasing most rapidly?(Putnam Exam Question) Answer: Making assumption that both the hands moves at constant angular velocities, the answer is ## \sqrt{7} .## But don't you think this assumption is somewhat doubtful and wrong?

Similar threads

  • · Replies 5 ·
Replies
5
Views
3K
  • · Replies 75 ·
3
Replies
75
Views
7K
  • · Replies 4 ·
Replies
4
Views
1K
  • · Replies 2 ·
Replies
2
Views
3K
  • · Replies 12 ·
Replies
12
Views
1K
Replies
3
Views
3K
  • · Replies 9 ·
Replies
9
Views
2K
  • · Replies 3 ·
Replies
3
Views
2K
  • · Replies 3 ·
Replies
3
Views
2K
  • · Replies 2 ·
Replies
2
Views
1K